0

当原始数据不可用时(这通常发生在我们从不同的科学论文中获取数据时),当只有中位数、IQR 和样本量可用时,如何执行 Mann-Whitney 分析来检验两个样本在统计上是否不同?

我想通过使用 R 指挥官来解决这个问题。

例如:

样本1:中位数1=2.5;IQR1=1.25-3.4;n1=4

样本2:中位数2=3.1;IQR2=2.25-6;n2=8

当进行 MW 时,它们在统计学上是否不同?

4

0 回答 0